LSAT and Law School Admissions Forum

Get expert LSAT preparation and law school admissions advice from PowerScore Test Preparation.

 Administrator
PowerScore Staff
  • PowerScore Staff
  • Posts: 8916
  • Joined: Feb 02, 2011
|
#24083
Complete Question Explanation

Flaw in the Reasoning—SN. The correct answer choice is (D)

The trustee in this stimulus presents a strange and flawed argument:

If attendance had been low, the museum would have gone bankrupt.

The local media seems to have contributed to the solid attendance.

Therefore, if it hadn’t been for the local media, the museum would be bankrupt.
  • Premise: ..... With media support ..... :arrow: ..... not bankrupt

    Conclusion: ..... With media support ..... :arrow: ..... bankrupt
We can see from the diagram above that the trustee’s conclusion is a mistaken negation. The museum needed good attendance, which kept it out of bankruptcy. Without the media, would the museum definitely have been doomed? There is no way to know how much the media contributed, nor what attendance was required to save the museum from bankruptcy. The flaw here is that the trustee presumes, without justification, that the media delivered the necessary attendance.

Answer choice (A): This answer choice describes a mistaken reversal, as opposed to the mistaken negation presented by the trustee.

Answer choice (B): There is no such presumption—the trustee only describes the media coverage as “extensive.”

Answer choice (C): The trustee makes no direct reference to reading, so this cannot be the flaw from the stimulus. The referenced local media may have been local television or radio crews.

Answer choice (D): This is the correct answer choice. Even if the media helped, it is possible that that attendance would have been high enough without the media contribution..

Answer choice (E): This answer choice describes the flaw of circular reasoning, which is different from the mistaken negation reflected by the trustee’s argument.
 bricbas
  • Posts: 9
  • Joined: Apr 20, 2016
|
#30339
I had a question about this one, and hopefully you can help me gain some clarity with it. When I initially did this question, I diagrammed it exactly as you said but I chose answer A. If I'm correct, essentially a mistaken negation and a mistaken reversal are inherently the same because if you find the contrapositive of a mistaken negation you get the mistaken reversal. Right? Would a correct answer choice EVER explain that a mistaken negation was a mistaken reversal since they are essentially one and the same?

But then I remembered reading that causality is basically exclusive. (If Media Coverage causes Record Attendance, on the LSAT Media Coverage is the ONLY thing that causes Record Attendance.) If this exclusiveness exists on the LSAT would it be safe to assume that, Although Attendance isn't causing the media coverage, it would still be sufficient for it conditionally. Right?

Anyway in my mind that would make this question a bit more clear. I can see why D is correct but I want to be absolutely certain A is wrong before I move on.
 Adam Tyson
PowerScore Staff
  • PowerScore Staff
  • Posts: 5153
  • Joined: Apr 14, 2011
|
#30403
Good question, bricbas, and I am going to do my best to supplement my esteemed colleague's official description of this argument. While I think it is largely correct, and that this question presents a very rare case of the test authors giving us both an MR and an MN as an answer choice and asking us to pick between them when they are, as you correctly noted, logically equivalent, there is another element to the argument that answer A overlooks.

The conditional portion of this stimulus is that if turnout was low, the museum would have gone bankrupt. We know further that the museum did not go bankrupt and we are also told that attendance was not low. That's perfect, a contrapositive. So what's the problem?

The problem is that this argument is not purely conditional, but is a hybrid of conditional logic and causal reasoning. The opening premise tells us that the media coverage contributed to the record turnout, and that is a type of causal claim. Unlike most causal claims on the LSAT, though, "contributed to" is a type of causal claim that actually allows for other, concurrent causes. In other words, the media coverage helped. Our author concludes that this help was essential - without it, we would not have had the record turnout - and that, in my opinion, is the real problem here. Just because one thing contributed to a certain effect does not prove that it was essential for that effect. Other causes may have been at work that were sufficient unto themselves.

It's impossible to talk about this argument without jumbling up conditional and causal language like I just did. I talked about contributing and then spoke about something being essential or sufficient. When that happens, I find that it's best to focus on the causal reasoning rather than the conditional reasoning, because causal reasoning is more direct and powerful and therefore easier to deal with head on.

So, the problem here is that there could be other causes involved and that media coverage, while helpful, was not necessary. That is answer D. Why not A, the Mistaken Reversal answer? Because it fails to address the causal aspect of the argument. It's a good answer, but D is an even better one because it brings in that causal element (where the purported cause is absent the effect can still be present).

This, to me, is a troubling question, because answer A is nearly as good as answer D. It should not matter whether we describe an MR or an MN, because they are the same darn thing. The only justification I can see for picking D over A is that injection of causality present by implication in D and absent from A.

Your analysis is good, and this question is confusing at best. I hope that helped clear things up rather than muddy the waters, but it is a muddy situation anyway. Good work! Let's hope we don't see more questions like this one!
 EL16
  • Posts: 45
  • Joined: Jul 06, 2017
|
#37821
Hi,

In the initial post about this question above, it has the following diagrammed:
  • Premise: ..... With media support ..... :arrow: ..... not bankrupt

    Conclusion: ..... With media support ..... :arrow: ..... bankrupt
I just want to make sure I understand that diagramming correctly: for the premise, it means that with media support there would be NO bankruptcy? And then for the conclusion, where it says "bankrupt" with a slash through it, is this a typo and it's NOT supposed to have a slash through it?

I just want to make sure I had my diagramming correct, and it doesn't seem to align with the above. My premise is the same (with media support :arrow: NOT bankrupt). But for the conclusion, I had NO media support :arrow: YES bankrupt, which seems to be different from the diagramming in the explanation above.

Thanks for the clarification,
Elana
 Francis O'Rourke
PowerScore Staff
  • PowerScore Staff
  • Posts: 471
  • Joined: Mar 10, 2017
|
#37915
Hi Elana, It does look like there was a typo in that post. The diagram should have read as follows:

Premise: ..... With media support ..... :arrow: ..... not bankrupt

Conclusion: ..... With media support ..... :arrow: ..... bankrupt
 LSAT2018
  • Posts: 242
  • Joined: Jan 10, 2018
|
#45216
Francis O'Rourke wrote:Hi Elana, It does look like there was a typo in that post. The diagram should have read as follows:

Premise: ..... With media support ..... :arrow: ..... not bankrupt

Conclusion: ..... With media support ..... :arrow: ..... bankrupt

Which sentence refers to your premise? I only saw the premise 'this coverage seems to have contributed to the record-breaking attendance it drew' as a causal statement.
So I wasn't able to see the Mistaken Negation here, only the conclusion that 'the museum would have gone bankrupt and closed permanently, so the museum could not have remained open had it not been for the coverage from the local media'
 Francis O'Rourke
PowerScore Staff
  • PowerScore Staff
  • Posts: 471
  • Joined: Mar 10, 2017
|
#45298
That requires some inference as the stimulus did not directly present the facts as a conditional statement as the administrator's post summarizes it.

We know that the media did extensively cover the exhibit (from the first sentence) and we know that the museum did not go bankrupt (from the final sentence).

Get the most out of your LSAT Prep Plus subscription.

Analyze and track your performance with our Testing and Analytics Package.